LSAT and Law School Admissions Forum

Get expert LSAT preparation and law school admissions advice from PowerScore Test Preparation.

User avatar
 Dave Killoran
PowerScore Staff
  • PowerScore Staff
  • Posts: 5852
  • Joined: Mar 25, 2011
|
#84897
Complete Question Explanation
(The complete setup for this game can be found here: lsat/viewtopic.php?t=8333)

The correct answer choice is (B)

If P is first, then Possibility #1 of the third rule is in play, but because P is first, we do not need to worry about the implications of the rule (because they are automatically met by P being first, and thus older than H and N). Thus, only the first and second rules play a role in determining which artifacts could be second.

The first rule eliminates H and J from being second. The second rule eliminates T from being second. Thus, four artifacts cannot be second (P, H, J, T), leaving two that can be second (F, N). Thus, answer choice (B) is correct.

Get the most out of your LSAT Prep Plus subscription.

Analyze and track your performance with our Testing and Analytics Package.